Ist die Dirac-Lagrange-Hermitesche?

  1. Ich wundere mich über die Dichte von Dirac Lagrange

    L = ψ ¯ ( ich γ μ μ + M ) ψ
    ist ein hermitescher Operator, da man bei komplexer Konjugation erhält
    L = ψ ( ich γ 0 γ μ γ 0 μ + M ) γ 0 ψ
    = ψ ¯ ( ich γ μ μ + M ) ψ L .

  2. Und sollte ein Lagrange immer hermitesch sein? Ich weiß, dass ein hermitescher Operator reelle Eigenwerte hat, was für einen Operator wünschenswert ist, der Observablen beschreibt. Aber hier ist die Lagrange-Funktion nicht wirklich eine Observable, da sie modulo einer totalen Ableitung bestimmt wird.

  3. Ich habe eine verwandte Frage auf der Website gefunden: wo das gesagt wird

    "Die Ableitung μ im Dirac Lagrange ist antihermitisch" ( Ist die Lagrange-Dichte in der Feldtheorie real? )

kann mir jemand zeigen wie man das demonstriert?

Der Dirac Lagrangian ist hermitesch, aber nicht real. Siehe hier: physical.stackexchange.com/questions/529496/…

Antworten (3)

( ψ γ 0 ψ ) = ψ γ 0 ψ
Weil γ 0 ist hermitesch. Auch,

( ψ ich γ 0 γ μ μ ψ ) = ich μ ψ γ μ γ 0 ψ = ich μ ψ ( γ 0 γ μ γ 0 ) γ 0 ψ = ich μ ψ γ 0 γ μ ψ = ich ψ γ 0 γ μ μ ψ + S u R F A C e T e R M
Für die zweite Zeile habe ich verwendet γ μ = γ 0 γ μ γ 0 und für die letzte Zeile habe ich nach Teilen integriert. Ich denke, Ihre Frage hängt von diesem Teil ab, da der letzte "Index", über den wir summieren, der Raumzeitindex ist X μ , dh Integration. Es ist der gleiche Grund, warum der quantenmechanische Impulsoperator P = ich X ist hermitesch.

Bearbeiten: Etwas, das ich übersehen habe, ist, dass die Spinoren auch Grassmann-Zahlen sind, also ist Vorsicht geboten. Insbesondere bedeutet dies, dass die Komponenten der Spinoren zufriedenstellend sind

( ψ ich ϕ J ) = ϕ J ψ ich

( mehr dazu hier ). Man vertauscht die Objekte schon beim Hermiteschen Konjugierten nach den Regeln der Matrizenalgebra, und man ist versucht, ein Minuszeichen einführen zu wollen, weil es Grassmann-Zahlen sind, aber das wäre überflüssig. Ausleihe aus der verlinkten math.se-Antwort :

( η ξ ) = [ ( A + ich B ) ( C + ich D ) ] = ( A C B D + ich B C + ich A D ) = C A D B ich C B ich D A = ( C ich D ) ( A ich B ) = ξ η

Im ersten Absatz erzwingen Sie die Bedingung der Hermitizität γ μ = γ 0 γ μ γ 0 kostet Sie das nicht relativistische Invarianz?

Im Prinzip kommt es auf die Wirkung an und nicht auf die Lagrange-Dichte, die streng genommen nicht als Observable betrachtet werden darf, da sie bis auf Randterme definiert ist. Was den von Ihnen betrachteten freien Lagrange betrifft, so ist er bis zu einem Grenzterm reell. Sie können es auch neu definieren, indem Sie den hermiteisch konjugierten Lagrangian, den Sie geschrieben haben, zum ursprünglichen addieren und die Hälfte des Ergebnisses nehmen. Diese neue Lagrange-Dichte ist real und entspricht der anfänglichen.

Die Lagrange-Dichte ist (immer) reell. Per Definition ist es also hermitesch.

Dies könnte besser sein, wenn Sie etwas mehr Exposition bereitstellen.
Diese Antwort ist falsch. Die Lagrange-Dichte ist nicht einmal immer hermitesch. Ist dies jedoch nicht der Fall, können Sie einen Term hinzufügen, der die Aktion (oder das EOM) nicht beeinflusst, wodurch es hermitesch wird. Dies ist jedoch nicht nur per Definition eine Lagrange-Dichte. Es muss für jedes L einzeln gezeigt werden.